Show that a continuous function with infinite “tails” with value $0$ and $f(0)=1$ must have a maximum.












0












$begingroup$



Let $f : R rightarrow R$ be continuous, where $f(0) = 1$, and
$$lim_{x rightarrow - infty}
f(x) = lim_{x rightarrow + infty}
f(x) = 0$$

Show: there is an $x^* in mathbb{R}$ such that $f(x^*) = max{f(x)| x in mathbb{R}}$




Intuitively this makes sense, we have some function that is above the $x$-axis for $x=0$, then as we look at its tails, we notice that for very large and for extremely small values the tails of the function go to zero. There must be some point where it is maximal, and this point must lie between $-infty$ and $+ infty$. I have been working on intermediate value theorem questions so far and I think this question is again an IVT-like question. I feel a bit lost when it comes down to "showing there exists a maximum". The intermediate value theorem tells you there must exist some intermediate point, but it does not say anything about it being a maximal point. Am I moving in the right direction or should I consider a different theorem (like the extreme value theorem $dots$ but then the question is, what is the interval because $[-infty, + infty]$ is just ridiculous)?





Edit: on a second note, this is probably more of an extreme value question, but I still am quite lost because it only works on a closed and compact/bounded interval $(-infty, + infty)$ is not bounded.










share|cite|improve this question











$endgroup$








  • 1




    $begingroup$
    Use definition of the limits, fix explicitly some large value of $x$ such that maximum is not attained beyond it. So now you have a bounded interval to work with.
    $endgroup$
    – AdditIdent
    Dec 8 '18 at 14:53








  • 2




    $begingroup$
    Extreme value theorem is the right way to go. The point of the exercise is that the limit to $pminfty$ conditions will make up for the fact that the interval is not bounded (please do note that $(-infty, infty) = mathbb{R}$ is closed, but it is not bounded/compact). Try using the limit conditions, with a particular choice of $varepsilon > 0$, to find a compact interval, the maximum over which must be the maximum over $mathbb{R}$.
    $endgroup$
    – Theo Bendit
    Dec 8 '18 at 14:53
















0












$begingroup$



Let $f : R rightarrow R$ be continuous, where $f(0) = 1$, and
$$lim_{x rightarrow - infty}
f(x) = lim_{x rightarrow + infty}
f(x) = 0$$

Show: there is an $x^* in mathbb{R}$ such that $f(x^*) = max{f(x)| x in mathbb{R}}$




Intuitively this makes sense, we have some function that is above the $x$-axis for $x=0$, then as we look at its tails, we notice that for very large and for extremely small values the tails of the function go to zero. There must be some point where it is maximal, and this point must lie between $-infty$ and $+ infty$. I have been working on intermediate value theorem questions so far and I think this question is again an IVT-like question. I feel a bit lost when it comes down to "showing there exists a maximum". The intermediate value theorem tells you there must exist some intermediate point, but it does not say anything about it being a maximal point. Am I moving in the right direction or should I consider a different theorem (like the extreme value theorem $dots$ but then the question is, what is the interval because $[-infty, + infty]$ is just ridiculous)?





Edit: on a second note, this is probably more of an extreme value question, but I still am quite lost because it only works on a closed and compact/bounded interval $(-infty, + infty)$ is not bounded.










share|cite|improve this question











$endgroup$








  • 1




    $begingroup$
    Use definition of the limits, fix explicitly some large value of $x$ such that maximum is not attained beyond it. So now you have a bounded interval to work with.
    $endgroup$
    – AdditIdent
    Dec 8 '18 at 14:53








  • 2




    $begingroup$
    Extreme value theorem is the right way to go. The point of the exercise is that the limit to $pminfty$ conditions will make up for the fact that the interval is not bounded (please do note that $(-infty, infty) = mathbb{R}$ is closed, but it is not bounded/compact). Try using the limit conditions, with a particular choice of $varepsilon > 0$, to find a compact interval, the maximum over which must be the maximum over $mathbb{R}$.
    $endgroup$
    – Theo Bendit
    Dec 8 '18 at 14:53














0












0








0





$begingroup$



Let $f : R rightarrow R$ be continuous, where $f(0) = 1$, and
$$lim_{x rightarrow - infty}
f(x) = lim_{x rightarrow + infty}
f(x) = 0$$

Show: there is an $x^* in mathbb{R}$ such that $f(x^*) = max{f(x)| x in mathbb{R}}$




Intuitively this makes sense, we have some function that is above the $x$-axis for $x=0$, then as we look at its tails, we notice that for very large and for extremely small values the tails of the function go to zero. There must be some point where it is maximal, and this point must lie between $-infty$ and $+ infty$. I have been working on intermediate value theorem questions so far and I think this question is again an IVT-like question. I feel a bit lost when it comes down to "showing there exists a maximum". The intermediate value theorem tells you there must exist some intermediate point, but it does not say anything about it being a maximal point. Am I moving in the right direction or should I consider a different theorem (like the extreme value theorem $dots$ but then the question is, what is the interval because $[-infty, + infty]$ is just ridiculous)?





Edit: on a second note, this is probably more of an extreme value question, but I still am quite lost because it only works on a closed and compact/bounded interval $(-infty, + infty)$ is not bounded.










share|cite|improve this question











$endgroup$





Let $f : R rightarrow R$ be continuous, where $f(0) = 1$, and
$$lim_{x rightarrow - infty}
f(x) = lim_{x rightarrow + infty}
f(x) = 0$$

Show: there is an $x^* in mathbb{R}$ such that $f(x^*) = max{f(x)| x in mathbb{R}}$




Intuitively this makes sense, we have some function that is above the $x$-axis for $x=0$, then as we look at its tails, we notice that for very large and for extremely small values the tails of the function go to zero. There must be some point where it is maximal, and this point must lie between $-infty$ and $+ infty$. I have been working on intermediate value theorem questions so far and I think this question is again an IVT-like question. I feel a bit lost when it comes down to "showing there exists a maximum". The intermediate value theorem tells you there must exist some intermediate point, but it does not say anything about it being a maximal point. Am I moving in the right direction or should I consider a different theorem (like the extreme value theorem $dots$ but then the question is, what is the interval because $[-infty, + infty]$ is just ridiculous)?





Edit: on a second note, this is probably more of an extreme value question, but I still am quite lost because it only works on a closed and compact/bounded interval $(-infty, + infty)$ is not bounded.







real-analysis






share|cite|improve this question















share|cite|improve this question













share|cite|improve this question




share|cite|improve this question








edited Dec 9 '18 at 10:42







Wesley Strik

















asked Dec 8 '18 at 14:45









Wesley StrikWesley Strik

1,741423




1,741423








  • 1




    $begingroup$
    Use definition of the limits, fix explicitly some large value of $x$ such that maximum is not attained beyond it. So now you have a bounded interval to work with.
    $endgroup$
    – AdditIdent
    Dec 8 '18 at 14:53








  • 2




    $begingroup$
    Extreme value theorem is the right way to go. The point of the exercise is that the limit to $pminfty$ conditions will make up for the fact that the interval is not bounded (please do note that $(-infty, infty) = mathbb{R}$ is closed, but it is not bounded/compact). Try using the limit conditions, with a particular choice of $varepsilon > 0$, to find a compact interval, the maximum over which must be the maximum over $mathbb{R}$.
    $endgroup$
    – Theo Bendit
    Dec 8 '18 at 14:53














  • 1




    $begingroup$
    Use definition of the limits, fix explicitly some large value of $x$ such that maximum is not attained beyond it. So now you have a bounded interval to work with.
    $endgroup$
    – AdditIdent
    Dec 8 '18 at 14:53








  • 2




    $begingroup$
    Extreme value theorem is the right way to go. The point of the exercise is that the limit to $pminfty$ conditions will make up for the fact that the interval is not bounded (please do note that $(-infty, infty) = mathbb{R}$ is closed, but it is not bounded/compact). Try using the limit conditions, with a particular choice of $varepsilon > 0$, to find a compact interval, the maximum over which must be the maximum over $mathbb{R}$.
    $endgroup$
    – Theo Bendit
    Dec 8 '18 at 14:53








1




1




$begingroup$
Use definition of the limits, fix explicitly some large value of $x$ such that maximum is not attained beyond it. So now you have a bounded interval to work with.
$endgroup$
– AdditIdent
Dec 8 '18 at 14:53






$begingroup$
Use definition of the limits, fix explicitly some large value of $x$ such that maximum is not attained beyond it. So now you have a bounded interval to work with.
$endgroup$
– AdditIdent
Dec 8 '18 at 14:53






2




2




$begingroup$
Extreme value theorem is the right way to go. The point of the exercise is that the limit to $pminfty$ conditions will make up for the fact that the interval is not bounded (please do note that $(-infty, infty) = mathbb{R}$ is closed, but it is not bounded/compact). Try using the limit conditions, with a particular choice of $varepsilon > 0$, to find a compact interval, the maximum over which must be the maximum over $mathbb{R}$.
$endgroup$
– Theo Bendit
Dec 8 '18 at 14:53




$begingroup$
Extreme value theorem is the right way to go. The point of the exercise is that the limit to $pminfty$ conditions will make up for the fact that the interval is not bounded (please do note that $(-infty, infty) = mathbb{R}$ is closed, but it is not bounded/compact). Try using the limit conditions, with a particular choice of $varepsilon > 0$, to find a compact interval, the maximum over which must be the maximum over $mathbb{R}$.
$endgroup$
– Theo Bendit
Dec 8 '18 at 14:53










3 Answers
3






active

oldest

votes


















2












$begingroup$

Find some $n>0$ such that $f(x)<0.5$ for $|x|>n$.



Now focus on $f$ restricted to closed and bounded (hence compact) interval $[-n,n]$.



A continuous function on a compact set takes a maximum so some $x^*in[-n,n]$ will exists with $f(x^*)=max({f(x)mid xin[-n,n]}$.



Then $f(x^*)geq f(0)=1>frac12$ so that also $f(x^*)>f(x)$ for every $xnotin[-n,n]$.



That means that $f(x^*)=max({f(x)mid xinmathbb R}$.






share|cite|improve this answer











$endgroup$





















    2












    $begingroup$

    By definition of the limits we can find $M$ such that $|x|>M implies f(x)<frac{1}{2}$. Now consider the restriction $f|_{[-M,M]}$. You can prove that the max of $f_{[-M,M]}$ is the max of (unrestricted) $f$.



    Another way to think about it is that the one-point compactification of $mathbb{R}$ is $S^1$, and the limit condition means you can extend $f$ continuously to $S^1$.






    share|cite|improve this answer









    $endgroup$





















      1












      $begingroup$

      Pick $x_+$ so large that $|f(x)| < frac{1}{10}$ for $x geq x_+$. And pick $x_-$ such that $|f(x)| < frac{1}{10}$ for $x leq x_-$ (we can do this since $f(x) to 0$ for $|x| to infty$. We may pick $x_- < x_+$. Now $f$ has a maximum in the interval $[x_-, x_+]$ and because $f(0)=1 > frac{1}{10}$ we know that this is a global maximum.






      share|cite|improve this answer









      $endgroup$













      • $begingroup$
        The function might oscillate a bit before it goes to zero. So we define it to be $frac{1}{x+1}sin(x) + frac{1}{x+1}$ and near $x=-1$ we define it differently piecewise so the two parts connect and it is continuous.
        $endgroup$
        – Wesley Strik
        Dec 9 '18 at 9:55












      • $begingroup$
        Then we don't attain a maximum at $1$ but it still satisfies the conditions.
        $endgroup$
        – Wesley Strik
        Dec 9 '18 at 9:57






      • 1




        $begingroup$
        I'm not saying that 1 is a maximum. Just that a maximum is not located outside $[x_-, x_+]$.
        $endgroup$
        – Martin
        Dec 9 '18 at 9:58











      Your Answer





      StackExchange.ifUsing("editor", function () {
      return StackExchange.using("mathjaxEditing", function () {
      StackExchange.MarkdownEditor.creationCallbacks.add(function (editor, postfix) {
      StackExchange.mathjaxEditing.prepareWmdForMathJax(editor, postfix, [["$", "$"], ["\\(","\\)"]]);
      });
      });
      }, "mathjax-editing");

      StackExchange.ready(function() {
      var channelOptions = {
      tags: "".split(" "),
      id: "69"
      };
      initTagRenderer("".split(" "), "".split(" "), channelOptions);

      StackExchange.using("externalEditor", function() {
      // Have to fire editor after snippets, if snippets enabled
      if (StackExchange.settings.snippets.snippetsEnabled) {
      StackExchange.using("snippets", function() {
      createEditor();
      });
      }
      else {
      createEditor();
      }
      });

      function createEditor() {
      StackExchange.prepareEditor({
      heartbeatType: 'answer',
      autoActivateHeartbeat: false,
      convertImagesToLinks: true,
      noModals: true,
      showLowRepImageUploadWarning: true,
      reputationToPostImages: 10,
      bindNavPrevention: true,
      postfix: "",
      imageUploader: {
      brandingHtml: "Powered by u003ca class="icon-imgur-white" href="https://imgur.com/"u003eu003c/au003e",
      contentPolicyHtml: "User contributions licensed under u003ca href="https://creativecommons.org/licenses/by-sa/3.0/"u003ecc by-sa 3.0 with attribution requiredu003c/au003e u003ca href="https://stackoverflow.com/legal/content-policy"u003e(content policy)u003c/au003e",
      allowUrls: true
      },
      noCode: true, onDemand: true,
      discardSelector: ".discard-answer"
      ,immediatelyShowMarkdownHelp:true
      });


      }
      });














      draft saved

      draft discarded


















      StackExchange.ready(
      function () {
      StackExchange.openid.initPostLogin('.new-post-login', 'https%3a%2f%2fmath.stackexchange.com%2fquestions%2f3031189%2fshow-that-a-continuous-function-with-infinite-tails-with-value-0-and-f0-1%23new-answer', 'question_page');
      }
      );

      Post as a guest















      Required, but never shown

























      3 Answers
      3






      active

      oldest

      votes








      3 Answers
      3






      active

      oldest

      votes









      active

      oldest

      votes






      active

      oldest

      votes









      2












      $begingroup$

      Find some $n>0$ such that $f(x)<0.5$ for $|x|>n$.



      Now focus on $f$ restricted to closed and bounded (hence compact) interval $[-n,n]$.



      A continuous function on a compact set takes a maximum so some $x^*in[-n,n]$ will exists with $f(x^*)=max({f(x)mid xin[-n,n]}$.



      Then $f(x^*)geq f(0)=1>frac12$ so that also $f(x^*)>f(x)$ for every $xnotin[-n,n]$.



      That means that $f(x^*)=max({f(x)mid xinmathbb R}$.






      share|cite|improve this answer











      $endgroup$


















        2












        $begingroup$

        Find some $n>0$ such that $f(x)<0.5$ for $|x|>n$.



        Now focus on $f$ restricted to closed and bounded (hence compact) interval $[-n,n]$.



        A continuous function on a compact set takes a maximum so some $x^*in[-n,n]$ will exists with $f(x^*)=max({f(x)mid xin[-n,n]}$.



        Then $f(x^*)geq f(0)=1>frac12$ so that also $f(x^*)>f(x)$ for every $xnotin[-n,n]$.



        That means that $f(x^*)=max({f(x)mid xinmathbb R}$.






        share|cite|improve this answer











        $endgroup$
















          2












          2








          2





          $begingroup$

          Find some $n>0$ such that $f(x)<0.5$ for $|x|>n$.



          Now focus on $f$ restricted to closed and bounded (hence compact) interval $[-n,n]$.



          A continuous function on a compact set takes a maximum so some $x^*in[-n,n]$ will exists with $f(x^*)=max({f(x)mid xin[-n,n]}$.



          Then $f(x^*)geq f(0)=1>frac12$ so that also $f(x^*)>f(x)$ for every $xnotin[-n,n]$.



          That means that $f(x^*)=max({f(x)mid xinmathbb R}$.






          share|cite|improve this answer











          $endgroup$



          Find some $n>0$ such that $f(x)<0.5$ for $|x|>n$.



          Now focus on $f$ restricted to closed and bounded (hence compact) interval $[-n,n]$.



          A continuous function on a compact set takes a maximum so some $x^*in[-n,n]$ will exists with $f(x^*)=max({f(x)mid xin[-n,n]}$.



          Then $f(x^*)geq f(0)=1>frac12$ so that also $f(x^*)>f(x)$ for every $xnotin[-n,n]$.



          That means that $f(x^*)=max({f(x)mid xinmathbb R}$.







          share|cite|improve this answer














          share|cite|improve this answer



          share|cite|improve this answer








          edited Dec 8 '18 at 15:01

























          answered Dec 8 '18 at 14:56









          drhabdrhab

          100k544130




          100k544130























              2












              $begingroup$

              By definition of the limits we can find $M$ such that $|x|>M implies f(x)<frac{1}{2}$. Now consider the restriction $f|_{[-M,M]}$. You can prove that the max of $f_{[-M,M]}$ is the max of (unrestricted) $f$.



              Another way to think about it is that the one-point compactification of $mathbb{R}$ is $S^1$, and the limit condition means you can extend $f$ continuously to $S^1$.






              share|cite|improve this answer









              $endgroup$


















                2












                $begingroup$

                By definition of the limits we can find $M$ such that $|x|>M implies f(x)<frac{1}{2}$. Now consider the restriction $f|_{[-M,M]}$. You can prove that the max of $f_{[-M,M]}$ is the max of (unrestricted) $f$.



                Another way to think about it is that the one-point compactification of $mathbb{R}$ is $S^1$, and the limit condition means you can extend $f$ continuously to $S^1$.






                share|cite|improve this answer









                $endgroup$
















                  2












                  2








                  2





                  $begingroup$

                  By definition of the limits we can find $M$ such that $|x|>M implies f(x)<frac{1}{2}$. Now consider the restriction $f|_{[-M,M]}$. You can prove that the max of $f_{[-M,M]}$ is the max of (unrestricted) $f$.



                  Another way to think about it is that the one-point compactification of $mathbb{R}$ is $S^1$, and the limit condition means you can extend $f$ continuously to $S^1$.






                  share|cite|improve this answer









                  $endgroup$



                  By definition of the limits we can find $M$ such that $|x|>M implies f(x)<frac{1}{2}$. Now consider the restriction $f|_{[-M,M]}$. You can prove that the max of $f_{[-M,M]}$ is the max of (unrestricted) $f$.



                  Another way to think about it is that the one-point compactification of $mathbb{R}$ is $S^1$, and the limit condition means you can extend $f$ continuously to $S^1$.







                  share|cite|improve this answer












                  share|cite|improve this answer



                  share|cite|improve this answer










                  answered Dec 8 '18 at 14:55









                  user25959user25959

                  1,573816




                  1,573816























                      1












                      $begingroup$

                      Pick $x_+$ so large that $|f(x)| < frac{1}{10}$ for $x geq x_+$. And pick $x_-$ such that $|f(x)| < frac{1}{10}$ for $x leq x_-$ (we can do this since $f(x) to 0$ for $|x| to infty$. We may pick $x_- < x_+$. Now $f$ has a maximum in the interval $[x_-, x_+]$ and because $f(0)=1 > frac{1}{10}$ we know that this is a global maximum.






                      share|cite|improve this answer









                      $endgroup$













                      • $begingroup$
                        The function might oscillate a bit before it goes to zero. So we define it to be $frac{1}{x+1}sin(x) + frac{1}{x+1}$ and near $x=-1$ we define it differently piecewise so the two parts connect and it is continuous.
                        $endgroup$
                        – Wesley Strik
                        Dec 9 '18 at 9:55












                      • $begingroup$
                        Then we don't attain a maximum at $1$ but it still satisfies the conditions.
                        $endgroup$
                        – Wesley Strik
                        Dec 9 '18 at 9:57






                      • 1




                        $begingroup$
                        I'm not saying that 1 is a maximum. Just that a maximum is not located outside $[x_-, x_+]$.
                        $endgroup$
                        – Martin
                        Dec 9 '18 at 9:58
















                      1












                      $begingroup$

                      Pick $x_+$ so large that $|f(x)| < frac{1}{10}$ for $x geq x_+$. And pick $x_-$ such that $|f(x)| < frac{1}{10}$ for $x leq x_-$ (we can do this since $f(x) to 0$ for $|x| to infty$. We may pick $x_- < x_+$. Now $f$ has a maximum in the interval $[x_-, x_+]$ and because $f(0)=1 > frac{1}{10}$ we know that this is a global maximum.






                      share|cite|improve this answer









                      $endgroup$













                      • $begingroup$
                        The function might oscillate a bit before it goes to zero. So we define it to be $frac{1}{x+1}sin(x) + frac{1}{x+1}$ and near $x=-1$ we define it differently piecewise so the two parts connect and it is continuous.
                        $endgroup$
                        – Wesley Strik
                        Dec 9 '18 at 9:55












                      • $begingroup$
                        Then we don't attain a maximum at $1$ but it still satisfies the conditions.
                        $endgroup$
                        – Wesley Strik
                        Dec 9 '18 at 9:57






                      • 1




                        $begingroup$
                        I'm not saying that 1 is a maximum. Just that a maximum is not located outside $[x_-, x_+]$.
                        $endgroup$
                        – Martin
                        Dec 9 '18 at 9:58














                      1












                      1








                      1





                      $begingroup$

                      Pick $x_+$ so large that $|f(x)| < frac{1}{10}$ for $x geq x_+$. And pick $x_-$ such that $|f(x)| < frac{1}{10}$ for $x leq x_-$ (we can do this since $f(x) to 0$ for $|x| to infty$. We may pick $x_- < x_+$. Now $f$ has a maximum in the interval $[x_-, x_+]$ and because $f(0)=1 > frac{1}{10}$ we know that this is a global maximum.






                      share|cite|improve this answer









                      $endgroup$



                      Pick $x_+$ so large that $|f(x)| < frac{1}{10}$ for $x geq x_+$. And pick $x_-$ such that $|f(x)| < frac{1}{10}$ for $x leq x_-$ (we can do this since $f(x) to 0$ for $|x| to infty$. We may pick $x_- < x_+$. Now $f$ has a maximum in the interval $[x_-, x_+]$ and because $f(0)=1 > frac{1}{10}$ we know that this is a global maximum.







                      share|cite|improve this answer












                      share|cite|improve this answer



                      share|cite|improve this answer










                      answered Dec 8 '18 at 14:59









                      MartinMartin

                      1,7541412




                      1,7541412












                      • $begingroup$
                        The function might oscillate a bit before it goes to zero. So we define it to be $frac{1}{x+1}sin(x) + frac{1}{x+1}$ and near $x=-1$ we define it differently piecewise so the two parts connect and it is continuous.
                        $endgroup$
                        – Wesley Strik
                        Dec 9 '18 at 9:55












                      • $begingroup$
                        Then we don't attain a maximum at $1$ but it still satisfies the conditions.
                        $endgroup$
                        – Wesley Strik
                        Dec 9 '18 at 9:57






                      • 1




                        $begingroup$
                        I'm not saying that 1 is a maximum. Just that a maximum is not located outside $[x_-, x_+]$.
                        $endgroup$
                        – Martin
                        Dec 9 '18 at 9:58


















                      • $begingroup$
                        The function might oscillate a bit before it goes to zero. So we define it to be $frac{1}{x+1}sin(x) + frac{1}{x+1}$ and near $x=-1$ we define it differently piecewise so the two parts connect and it is continuous.
                        $endgroup$
                        – Wesley Strik
                        Dec 9 '18 at 9:55












                      • $begingroup$
                        Then we don't attain a maximum at $1$ but it still satisfies the conditions.
                        $endgroup$
                        – Wesley Strik
                        Dec 9 '18 at 9:57






                      • 1




                        $begingroup$
                        I'm not saying that 1 is a maximum. Just that a maximum is not located outside $[x_-, x_+]$.
                        $endgroup$
                        – Martin
                        Dec 9 '18 at 9:58
















                      $begingroup$
                      The function might oscillate a bit before it goes to zero. So we define it to be $frac{1}{x+1}sin(x) + frac{1}{x+1}$ and near $x=-1$ we define it differently piecewise so the two parts connect and it is continuous.
                      $endgroup$
                      – Wesley Strik
                      Dec 9 '18 at 9:55






                      $begingroup$
                      The function might oscillate a bit before it goes to zero. So we define it to be $frac{1}{x+1}sin(x) + frac{1}{x+1}$ and near $x=-1$ we define it differently piecewise so the two parts connect and it is continuous.
                      $endgroup$
                      – Wesley Strik
                      Dec 9 '18 at 9:55














                      $begingroup$
                      Then we don't attain a maximum at $1$ but it still satisfies the conditions.
                      $endgroup$
                      – Wesley Strik
                      Dec 9 '18 at 9:57




                      $begingroup$
                      Then we don't attain a maximum at $1$ but it still satisfies the conditions.
                      $endgroup$
                      – Wesley Strik
                      Dec 9 '18 at 9:57




                      1




                      1




                      $begingroup$
                      I'm not saying that 1 is a maximum. Just that a maximum is not located outside $[x_-, x_+]$.
                      $endgroup$
                      – Martin
                      Dec 9 '18 at 9:58




                      $begingroup$
                      I'm not saying that 1 is a maximum. Just that a maximum is not located outside $[x_-, x_+]$.
                      $endgroup$
                      – Martin
                      Dec 9 '18 at 9:58


















                      draft saved

                      draft discarded




















































                      Thanks for contributing an answer to Mathematics Stack Exchange!


                      • Please be sure to answer the question. Provide details and share your research!

                      But avoid



                      • Asking for help, clarification, or responding to other answers.

                      • Making statements based on opinion; back them up with references or personal experience.


                      Use MathJax to format equations. MathJax reference.


                      To learn more, see our tips on writing great answers.




                      draft saved


                      draft discarded














                      StackExchange.ready(
                      function () {
                      StackExchange.openid.initPostLogin('.new-post-login', 'https%3a%2f%2fmath.stackexchange.com%2fquestions%2f3031189%2fshow-that-a-continuous-function-with-infinite-tails-with-value-0-and-f0-1%23new-answer', 'question_page');
                      }
                      );

                      Post as a guest















                      Required, but never shown





















































                      Required, but never shown














                      Required, but never shown












                      Required, but never shown







                      Required, but never shown

































                      Required, but never shown














                      Required, but never shown












                      Required, but never shown







                      Required, but never shown







                      Popular posts from this blog

                      Quarter-circle Tiles

                      build a pushdown automaton that recognizes the reverse language of a given pushdown automaton?

                      Mont Emei